Derivación de la densidad lagrangiana del campo electromagnético

Al considerar el campo EM relativista (especial), entiendo que asumiendo una densidad lagrangiana de la forma

L = 1 4 F m v F m v + 1 C j m A m

y siguiendo las ecuaciones de Euler-Lagrange recupera las ecuaciones de Maxwell.

¿Existe una derivación de primeros principios de este Lagrangiano? ¡Una referencia o explicación sería muy apreciada!

¿De qué principios quieres partir?

Respuestas (5)

Abstracto

A continuación, probaremos que una densidad lagrangiana compatible para el campo electromagnético en el espacio vacío es

(045) L mi metro = ϵ 0 | | mi | | 2 C 2 | | B | | 2 2 ρ ϕ + j A
es decir, las ecuaciones de Euler-Langrange producidas a partir de este Lagrangiano son las ecuaciones de Maxwell para el campo electromagnético.

Esta densidad lagrangiana se obtiene mediante un procedimiento de prueba y error (1) , no por adivinanzas.

1. Introducción

Las ecuaciones diferenciales de Maxwell del campo electromagnético en el espacio vacío son

(001a) × mi = B t (001b) × B = m 0 j + 1 C 2 mi t (001c) mi = ρ ϵ 0 (001d) B = 0
dónde mi = vector de intensidad de campo eléctrico, B = vector de densidad de flujo magnético, ρ = densidad de carga eléctrica, j = vector de densidad de corriente eléctrica. Todas las cantidades son funciones de las tres coordenadas espaciales. ( X 1 , X 2 , X 3 ) ( X , y , z ) y tiempo t X 4 .

De la ecuación (001d) el vector de flujo magnético B puede expresarse como el rotacional de un potencial vectorial A

(002) B = × A
y de (002) la ecuación (001a) da
(003) × ( mi + A t ) = 0
por lo que el término entre paréntesis se puede expresar como el gradiente de una función escalar
mi + A t = ϕ
eso es
(004) mi = ϕ A t
Así que las seis variables escalares, las componentes de los vectores mi y B , se puede expresar como funciones de 4 variables escalares, el potencial escalar ϕ y tres componentes del vector potencial A .

Insertar las expresiones de mi y B , ecuaciones (002) y (004) respectivamente, en las ecuaciones (001b) y (001c) tenemos

(005) × ( × A ) = m 0 j + 1 C 2 t ( ϕ A t )

y
(006) 2 ϕ t ( A ) = ρ ϵ 0
Dado que
(007) × ( × A ) = ( A ) 2 A
la ecuación (005) produce
(008) 1 C 2 2 A t 2 2 A + ( A + 1 C 2 ϕ t ) = m 0 j

2. Las ecuaciones de Euler-Lagrange del campo EM

Ahora, nuestra tarea principal es encontrar una densidad lagrangiana L , función de las cuatro ''coordenadas de campo'' y sus derivadas de primer orden

(009) L = L ( η ȷ , η ȷ , η ȷ ) ( ȷ = 1 , 2 , 3 , 4 )
tal que las cuatro ecuaciones de campo electromagnético escalares (006) y (008) se derivan de las ecuaciones de Lagrange
(010) t [ L ( η ȷ t ) ] + k = 1 k = 3 X k [ L ( η ȷ X k ) ] L η ȷ = 0 , ( ȷ = 1 , 2 , 3 , 4 )
simplificado en notación a
(011) t ( L η ȷ ) + [ L ( η ȷ ) ] L η ȷ = 0 , ( ȷ = 1 , 2 , 3 , 4 )

Aquí la densidad lagrangiana L es una función de

  1. las cuatro ''coordenadas de campo''

(012.1) η 1 = A 1 ( X 1 , X 2 , X 3 , t ) (012.2) η 2 = A 2 ( X 1 , X 2 , X 3 , t ) (012.3) η 3 = A 3 ( X 1 , X 2 , X 3 , t ) (012.4) η 4 = ϕ ( X 1 , X 2 , X 3 , t )

  1. sus derivados temporales

(013.1) η 1 η 1 t = A 1 t A 1 (013.2) η 2 η 2 t = A 2 t A 2 (013.3) η 3 η 3 t = A 3 t A 3 (013.4) η 4 η 4 t = ϕ t ϕ

y

  1. sus gradientes

(014) η 1 = A 1 , η 2 = A 2 , η 3 = A 3 , η 4 = ϕ

Expresamos las ecuaciones (006) y (008) en formas que son similares a las ecuaciones de Lagrange (011)

(015) t ( A ) + ( ϕ ) ( ρ ϵ 0 ) = 0
y
(016) t ( A k t + ϕ X k ) + [ C 2 ( A X k A k ) ] j k ϵ 0 = 0
La ecuación de Lagrange (011) para ȷ = 4 , eso es para η 4 = ϕ , es
(017) t ( L ϕ ) + [ L ( ϕ ) ] L ϕ = 0

Comparando las ecuaciones (015) y (017), notamos que la primera podría derivarse de la segunda si

(018) L ϕ = A , L ( ϕ ) = ϕ , L ϕ = ρ ϵ 0

de modo que la densidad lagrangiana L debe contener respectivamente los términos
(019) L α 1 ( A ) ϕ , L α 2 1 2 ϕ 2 , L α 3 ρ ϕ ϵ 0
y en consecuencia su suma
(020) L α = L α 1 + L α 2 + L α 3 = ( A ) ϕ + 1 2 ϕ 2 ρ ϕ ϵ 0

Suponemos que una densidad lagrangiana apropiada L sería de la forma

(021) L = L α + L β
y desde L α produce la ecuación (015), esperamos que L β , por determinar, producirá las ecuaciones (016). Esta expectativa sería correcta si las ecuaciones (015) y (016) estuvieran desacopladas, por ejemplo si la primera contiene ϕ -términos solamente y el segundo A -términos solamente. Pero aquí este no es el caso: L α como que contiene A -términos participarían en la producción de ecuaciones (016) y además L β participaría en la producción de la ecuación (015), posiblemente destruyendo mutuamente la producción de las ecuaciones como esperábamos. Pero aquí seguimos un procedimiento de prueba y error, que nos llevará a la respuesta correcta como veremos a continuación.

Ahora, las ecuaciones de Lagrange (011) para ȷ = k = 1 , 2 , 3 , eso es para η k = A k , son

(022) t ( L A k ) + [ L ( A k ) ] L A k = 0

Comparando las ecuaciones (016) y (022), notamos que la primera podría derivarse de la segunda si

(023) L A k = A k + ϕ X k , L ( A k ) = C 2 ( A X k A k ) , L A k = j k ϵ 0

De la primera de las ecuaciones (023) el L β parte de la densidad de Lagrange L debe contener los términos

(024) 1 2 A k 2 + ϕ X k A k , k = 1 , 2 , 3
y por lo tanto su suma con respecto a k
(025) L β 1 1 2 A ˙ 2 + ϕ A ˙

A partir de la 2ª de las ecuaciones (023) el L β parte de la densidad de Lagrange L debe contener los términos

(026) 1 2 C 2 [ A X k A k A k 2 ] , k = 1 , 2 , 3
y por lo tanto su suma con respecto a k
(027) L β 2 1 2 C 2 k = 1 k = 3 [ A X k A k A k 2 ]
De la 3ra de las ecuaciones (023) el L β parte de la densidad de Lagrange L debe contener los términos
(028) j k A k ϵ 0 , k = 1 , 2 , 3
y por lo tanto su suma con respecto a k
(029) L β 3 j A ϵ 0

De las ecuaciones (025), (027) y (029) la L β parte de la densidad de Lagrange L es

(030) L β = L β 1 + L β 2 + L β 3 = 1 2 A ˙ 2 + ϕ A ˙ + 1 2 C 2 k = 1 k = 3 [ A X k A k A k 2 ] + j A ϵ 0

Finalmente, de las expresiones (020) y (030) para las densidades L α , L β la densidad de Lagrange L = L α + L β es

(031) L = L α + L β = ( A ) ϕ + 1 2 ϕ 2 ρ ϕ ϵ 0 + 1 2 A ˙ 2 + ϕ A ˙ + 1 2 C 2 k = 1 k = 3 [ A X k A k A k 2 ] + j A ϵ 0 (esta es una densidad de Lagrange incorrecta)

3. Error-Prueba-Éxito final

Inserción de esta expresión de densidad de Lagrange en la ecuación de Lagrange con respecto a ϕ , que es la ecuación (017), no produce la ecuación (006) pero

(032) 2 ϕ t ( 2 A ) = ρ ϵ 0 , ( equivocado )
La aparición de un extra ( A ) se debe al término ( ϕ A ˙ ) de L β y por eso la densidad de Lagrange dada por la ecuación (031) no es adecuada.

Para resolver este problema debemos mirar (015), es decir (006), desde un punto de vista diferente como sigue

(033) ( ϕ + A ˙ ) ( ρ ϵ 0 ) = 0

Comparando las ecuaciones (033) y (017), notamos que la primera podría derivarse de la segunda si en lugar de (018) tenemos

(034) L ϕ = 0 , L ( ϕ ) = ϕ + A ˙ , L ϕ = ρ ϵ 0

entonces en lugar de (019) y (020) respectivamente las ecuaciones
(035) L α 1 0 , L α 2 1 2 ϕ 2 + ϕ A ˙ , L α 3 = L α 3 ρ ϕ ϵ 0
(036) L α = L α 1 + L α 2 + L α 3 = 1 2 ϕ 2 + ϕ A ˙ ρ ϕ ϵ 0
Ahora, es necesario omitir de L β 1 , ecuación (025), el segundo término ( ϕ A ˙ ) ya que aparece en L α 2 , véase la segunda de las ecuaciones anteriores (035).

Entonces tenemos en lugar de (025)

(037) L β 1 1 2 A ˙ 2
mientras L β 2 , L β 3 permanecen sin cambios como en las ecuaciones (027) y (029)
(038) L β 2 = L β 2 1 2 C 2 k = 1 k = 3 [ A X k A k A k 2 ] (039) L β 3 = L β 3 j A ϵ 0

En lugar de (030)

(040) L β = L β 1 + L β 2 + L β 3 = 1 2 A ˙ 2 + 1 2 C 2 k = 1 k = 3 [ A X k A k A k 2 ] + j A ϵ 0
y finalmente para la nueva densidad lagrangiana tenemos en lugar de (031)

(041) L = L α + L β = 1 2 ϕ 2 + ϕ A ˙ ρ ϕ ϵ 0 + 1 2 A ˙ 2 + 1 2 C 2 k = 1 k = 3 [ A X k A k A k 2 ] + j A ϵ 0

Densidad L de (041) se obtiene a partir de la densidad L de (031) si omitimos el término ( A ) ϕ . Entonces L es independiente de ϕ .

En las siguientes ecuaciones, la llave sobre los 3 términos de la izquierda agrupa esa parte de la densidad L que esencialmente participa en la producción de la ecuación electromagnética (006) a partir de la ecuación de Lagrange con respecto a ϕ , ecuación (017), mientras que la llave debajo de los 4 términos de la derecha agrupa esa parte de la densidad L que esencialmente participa en la producción de las ecuaciones electromagnéticas (008) a partir de las ecuaciones de Lagrange con respecto a A 1 , A 2 , A 3 , ecuación (022).

L = 1 2 ϕ 2 ρ ϕ ϵ 0 + ϕ A ˙ con respecto a  ϕ + 1 2 A ˙ 2 + 1 2 C 2 k = 1 k = 3 [ A X k A k A k 2 ] + j A ϵ 0

L = 1 2 ϕ 2 ρ ϕ ϵ 0 + ϕ A ˙ + 1 2 A ˙ 2 + 1 2 C 2 k = 1 k = 3 [ A X k A k A k 2 ] + j A ϵ 0 con respecto a  A

Tenga en cuenta el término común ( ϕ A ˙ ) .

Reordenando los términos en la expresión (041) de la densidad L tenemos

L = 1 2 A ˙ 2 + 1 2 ϕ 2 + ϕ A ˙ 1 2 ϕ A t 2 1 2 C 2 k = 1 k = 3 [ A k 2 A X k A k ] × A 2 + 1 ϵ 0 ( ρ ϕ + j A )
(042)

eso es

(043) L = 1 2 | | ϕ A t | | 2 1 2 C 2 | | × A | | 2 + 1 ϵ 0 ( ρ ϕ + j A )
o
(044) L = | | mi | | 2 C 2 | | B | | 2 2 + 1 ϵ 0 ( ρ ϕ + j A )

Ahora bien, si la densidad L debe tener dimensiones de energía por unidad de volumen que definimos L mi metro = ϵ 0 L entonces

(045) L mi metro = ϵ 0 | | mi | | 2 C 2 | | B | | 2 2 ρ ϕ + j A
teniendo en cuenta que
(046a) mi 2 = ϕ A t 2 = A ˙ 2 + ϕ 2 + 2 ( ϕ A ˙ ) (046b) B 2 = × A 2 = k = 1 k = 3 [ A k 2 A X k A k ]

el escalar ( | | mi | | 2 C 2 | | B | | 2 ) es una de las dos invariantes de Lorentz (2) del campo (la otra es mi B ) esencialmente igual a una constante veces mi m v mi m v , dónde mi m v el tensor de campo antisimétrico (2) .

Por otro lado, el escalar ( ρ ϕ + j A ) es esencialmente el producto interno j m A m en el espacio de Minkowski de dos 4 vectores: la densidad de 4 corrientes j m = ( C ρ , j ) y el 4-potencial A m = ( ϕ / C , A ) , un escalar invariante de Lorentz también.

Entonces, la densidad de Lagrange L mi metro en la ecuación (045) es invariante de Lorentz.


(1) Mediante un procedimiento de prueba y error, encontré el Lagrangiano en un caso más difícil y complicado: vea mi respuesta como usuario82794 aquí Obtenga el Lagrangiano del sistema de ecuación acoplada

(2) Siguiendo a W.Rindler en "Introducción a la Relatividad Especial" Ed.1982, este tensor se deriva en la ecuación (38.15)

(38.15) mi m v = [ 0 mi 1 mi 2 mi 3 mi 1 0 C B 3 C B 2 mi 2 C B 3 0 C B 1 mi 3 C B 2 C B 1 0 ] entonces mi m v = [ 0 mi 1 mi 2 mi 3 mi 1 0 C B 3 C B 2 mi 2 C B 3 0 C B 1 mi 3 C B 2 C B 1 0 ]
que al hacer los reemplazos (dualidad) mi C B y C B mi rendimientos
(39.05) B m v = [ 0 C B 1 C B 2 C B 3 C B 1 0 mi 3 mi 2 C B 2 C mi 3 0 mi 1 C B 3 mi 2 mi 1 0 ] entonces B m v = [ 0 C B 1 C B 2 C B 3 C B 1 0 mi 3 mi 2 C B 2 C mi 3 0 mi 1 C B 3 mi 2 mi 1 0 ]
Las dos invariantes de mi m v -inmediatamente reconocibles como tales por su modo de formación- se puede expresar de la siguiente manera:
(39.06) X = 1 2 mi m v mi m v = 1 2 B m v B m v = C 2 | | B | | 2 | | mi | | 2 (39.07) Y = 1 4 B m v mi m v = C B mi

¿Cuál es la diferencia entre "ensayo y error" y "adivinar"?
Hermoso látex
Un profesor, llamado Bahman Zohuri , del Departamento de Ingeniería Eléctrica e Informática de la Universidad de Nuevo México, publicó el 30 de enero de 2019 un artículo en formato pdf titulado "Derivar la densidad lagrangiana de un campo electromagnético" con una copia exacta de esta respuesta. Aquí en. Es una pena que en sus referencias ni siquiera una palabra sobre PhysicsStackExchange y mi respuesta. Enlace: Derivación de la densidad lagrangiana de un campo electromagnético
Es difícil creer que un profesor de una universidad "reputada" plagió tu mundo sin siquiera mencionarlo. Lo que es más sorprendente es que un editor de "reputación" lo publicó, repugnante por decir lo menos.
@renormalizedQuanta: ...de hecho, es difícil de creer...
¡Esta respuesta es genial! (+++1)
@Frobenius, sobre ese copiar y pegar, creo que sería mejor que alguien se comunique con Springer e informe ese plagio. Las revistas prestigiosas suelen acusar a los profesores de plagio. Estoy bastante seguro de que esta no es la primera vez que lo hace y no será la última vez.
En el momento en que pensé que deberías publicar esto, ¡vi que un profesor había (ab)usado esto!
@Deschele Schilder: ... ¿qué puedo decir sobre esto? Como puede ver, publique este comentario el 4 de junio de 19 y no respondí. Estoy seguro de que el profesor de una universidad "reputada" y el editor "reputado" saben sobre esto. Por cierto, siempre es un placer ver comentarios como este aquí Un argumento matemáticamente ilógico en la derivación de la ecuación de Hamilton en Goldstein .
En la ecuación (016), ¿cómo m 0 j ir a j k ϵ 0 ?
@moboDawn_φ: Bienvenido a PSE. Porque
C 2 ( 008 ) = ( 016 ) C 2 m 0 j = j ϵ 0
¡Gracias por esta increíble respuesta y trabajo!
@abu_bua: Bienvenido. Sugiero echar un vistazo también aquí ¿ Por qué tratar el campo escalar complejo y su conjugado complejo como dos campos diferentes? para la densidad lagrangiana de la ecuación de Schroedinger.
@Frobenius, ¿ha informado al editor/editor?
@ziyuang: Gracias por su atención. No, no he informado. No creo que valga la pena hacerlo.
@Frobenius ¡Buena respuesta! Aunque un comentario! Las ecuaciones de Maxwell que escribiste no son ecuaciones de Maxwell en el espacio vacío. Están en presencia de cargas y corrientes ( ρ , j 0 ).

En última instancia, el razonamiento debe ser que (como usted indicó) debe construirse de modo que las ecuaciones de Euler-Lagrange sean las ecuaciones de Maxwell. Entonces, en cierto sentido, debe adivinar el Lagrangiano que produce esto, como se hace aquí, por ejemplo.

Sin embargo, puede obtener alguna orientación del hecho de que necesitamos construir un Lagrangiano para un campo sin autointeracción sin masa. Así que necesitamos una combinación invariante de calibre y lorentz del potencial de 4 vectores que solo tiene un término cinético (cuadrático en derivadas de los campos). Entonces no te quedan muchas opciones aparte de F m v F m v . El término fuente es entonces trivial de agregar si es necesario.

Qué pasa ϵ m v σ τ F m v F σ τ ?
Bueno, dije "no me quedan muchas opciones" y, de hecho, la combinación que anotas también se ajustaría a mis criterios, al igual que det ( F ) , pero también se puede probar la opción más simple primero y resulta ser correcta. Observo que la combinación que das es un pseudoescalar. ¿Alguien puede pensar si hay una razón por la que eso no estaría permitido?
@MistakeInk - ϵ m v ρ σ F m v F ρ σ es un buen término candidato para el lagragiano, es solo que es una derivada total, por lo que no afecta el EOM clásico y desaparece en la teoría de perturbaciones. Sin embargo, todavía tiene algunas consecuencias: consulte en.wikipedia.org/wiki/CP_violation#Strong_CP_problem . Como para D mi t ( F ) No creo que este término sea renormalizable ya que es igual a mi t r registro F que podría expandirse sobre algún valor de campo de fondo y obtener potencias arbitrariamente altas de la intensidad del campo.
Sin embargo, obtiene términos de la forma registro t r ( k 2 + F m v 2 ) al calcular la acción efectiva en presencia de un campo indicador de campo de fondo. Véase el capítulo 16 de Peskin.
@DJBunk Tienes razón en que det ( F v m ) no es renormalizable, por lo que no funcionaría, pero solo es cuartico en F y no contiene poderes arbitrariamente altos. Tampoco es invariante de Lorentz.
@DJBunk Un distinto de cero ϵ F F El término se ha realizado en realidad en sistemas experimentales reales: aisladores topológicos 3D. No afecta la física general, pero da lugar a modos de borde sin espacios topológicamente protegidos en el límite del sistema.

Estoy casi 100% seguro de que el Lagrangiano es una suposición de la teoría. No se puede derivar. No tengo ninguna referencia para este reclamo. Solo sé que de cada curso que me han enseñado y cada libro que he leído, el Lagrangiano (suponiendo que se esté usando) es donde comienzas. Es el "primer principio" en este caso.

Gracias, supongo que solo había visto lagrangianos de la mecánica, donde son naturalmente de la forma L = T V y así lo que yo estaba llamando "derivable".
Tampoco veo cómo ese Lagrangiano es "derivable". Por supuesto, lo escribimos como tal para que las ecuaciones de Euler-Lagrange nos den las ecuaciones clásicas de movimiento. Pero, no lo consideraría una derivación. Realmente es solo sustituir una suposición por otra.

Puede encontrar la respuesta en el libro "Differential Geometry and Lie Groups for Physicists" de Marian Fecko.

En lenguaje geométrico, una acción de un campo F Ω pag ( METRO ) en alguna variedad riemanniana n-dim ( METRO , gramo ) debe entenderse como un 'producto interno'

METRO F gramo F ,
dónde pag < norte , y gramo es el operador estrella de hodge, es decir
gramo : Ω pag ( METRO ) Ω norte pag ( METRO )
por lo que la acción es difeomorfismo invariante y su densidad es un escalar.

Por ejemplo, la acción de un campo escalar libre toma esta forma:

S [ ϕ ] = METRO d ϕ gramo d ϕ + metro 2 METRO ϕ gramo ϕ = METRO | gramo | d norte X { m ϕ m ϕ + metro ϕ 2 2 } .
Cuando la variedad 'worldsheet' es 1-dim, solo los campos posibles son 1-formas. Se puede considerar que una acción toma la siguiente forma
R A m d X m d s d s
dónde A = A s d s = A m d X m d s d s es una forma 1 en la línea del mundo, cuyo campo dual hodge star no está definido.

Puede usar las simetrías de E&M para mostrar que esencialmente solo hay un candidato razonable que debe verificarse:

La acción que buscamos debe ser invariante de Lorentz, invariante de calibre, invariante de paridad e inversión de tiempo, y no más de segundo orden en derivadas. El único candidato es [la acción de Maxwell]. [Srednicki QFT pág. 334.]